Derivative of Secant: Find dy/dx

  • Thread starter Thread starter Karol
  • Start date Start date
  • Tags Tags
    Derivative Secant
Click For Summary
SUMMARY

The discussion focuses on finding the derivative of the function \( y = \sec^2(5x) \). The correct derivative is calculated using the chain rule and the derivative of secant, resulting in \( y' = 10\tan(5x)\sec^2(5x) \). The solution provided by the user is confirmed as accurate, demonstrating a clear understanding of the differentiation process involving trigonometric functions.

PREREQUISITES
  • Understanding of trigonometric functions, specifically secant and tangent.
  • Knowledge of the chain rule in calculus.
  • Familiarity with differentiation techniques for composite functions.
  • Ability to manipulate and simplify trigonometric expressions.
NEXT STEPS
  • Study the chain rule in depth, focusing on its application in differentiating composite functions.
  • Learn about the derivatives of other trigonometric functions, such as cosecant and cotangent.
  • Explore advanced differentiation techniques, including implicit differentiation.
  • Practice solving similar problems involving derivatives of trigonometric functions.
USEFUL FOR

Students studying calculus, particularly those focusing on differentiation of trigonometric functions, and educators looking for examples of applying the chain rule in calculus problems.

Karol
Messages
1,380
Reaction score
22

Homework Statement


Find dy/dx for ##~y=\sec^2(5x)##

Homework Equations


Secant and it's derivative:
$$\sec\,x=\frac{1}{\cos\,x}$$
$$\sec'\,x=\tan\,x\cdot\sec\,x$$

The Attempt at a Solution


$$y=\sec^2(5x)~\rightarrow~y'=2\cdot 5 \cdot \sec(5x)\tan(5x)\sec(5x)=10\tan(5x)\sec^2(5x)$$
The answer should be:
$$y'=-\frac{2x+1}{2}\frac{1}{\sqrt{(x^2+x-1)^3}}$$
 
Physics news on Phys.org
Your solution is correct to the given problem.
 
Thank you very much jambaugh
 
Question: A clock's minute hand has length 4 and its hour hand has length 3. What is the distance between the tips at the moment when it is increasing most rapidly?(Putnam Exam Question) Answer: Making assumption that both the hands moves at constant angular velocities, the answer is ## \sqrt{7} .## But don't you think this assumption is somewhat doubtful and wrong?

Similar threads

Replies
6
Views
2K
  • · Replies 4 ·
Replies
4
Views
2K
  • · Replies 10 ·
Replies
10
Views
2K
  • · Replies 5 ·
Replies
5
Views
3K
  • · Replies 8 ·
Replies
8
Views
2K
  • · Replies 15 ·
Replies
15
Views
2K
Replies
2
Views
1K
  • · Replies 6 ·
Replies
6
Views
2K
Replies
11
Views
2K
  • · Replies 6 ·
Replies
6
Views
2K